2016 AMC 8 Problems/Problem 22

Revision as of 11:32, 23 November 2016 by Reaganchoi (talk | contribs) (Created page with "Rectangle <math>DEFA</math> below is a <math>3 \times 4</math> rectangle with <math>DC=CB=BA</math> The area of the "bat wings" is [asy] draw((0,0)--(3,0)--(3,4)--(0,4)--(0,0)...")
(diff) ← Older revision | Latest revision (diff) | Newer revision → (diff)

Rectangle $DEFA$ below is a $3 \times 4$ rectangle with $DC=CB=BA$ The area of the "bat wings" is [asy] draw((0,0)--(3,0)--(3,4)--(0,4)--(0,0)--(2,4)--(3,0)); draw((3,0)--(1,4)--(0,0)); fill((0,0)--(1,4)--(1.5,3)--(0,0), black); fill((3,0)--(2,4)--(1.5,3)--(3,0), black); [/asy]

$\textbf{(A) }2\qquad\textbf{(B) }2 \frac{1}{2}\qquad\textbf{(C) }3\qquad\textbf{(D) }3 \frac{1}{2}\qquad \textbf{(E) }5$

Solution

This problem needs a solution. If you have a solution for it, please help us out by adding it.

2016 AMC 8 (ProblemsAnswer KeyResources)
Preceded by
Problem 12
Followed by
Problem 14
1 2 3 4 5 6 7 8 9 10 11 12 13 14 15 16 17 18 19 20 21 22 23 24 25
All AJHSME/AMC 8 Problems and Solutions

The problems on this page are copyrighted by the Mathematical Association of America's American Mathematics Competitions. AMC logo.png